You are on page 1of 9

Machine Translated by Google

Consultez les discussions, les statistiques et les profils des auteurs de cette publication surÿ: https://www.researchgate.net/publication/331628227

Une alternative à la théorie électromagnétique classique

Prépublication · Mars 2015

DOIÿ: 10.13140/RG.2.2.16250.75208

CITATIONS LIT

3 654

1 auteur :

Diego Saa
Escuela Politécnica Nacional

14 PUBLICATIONS 18 CITATIONS

VOIR LE PROFIL

Certains des auteurs de cette publication travaillent également sur ces projets connexesÿ:

Logique mathématique Voir le projet

Physique Voir le projet

Tout le contenu suivant cette page a été téléchargé par Diego Saa le 09 mars 2019.

L'utilisateur a demandé l'amélioration du fichier téléchargé.


Machine Translated by Google
MARS 2015 1

Une alternative à l'électromagnétique classique


La théorie
´
Diego Saa, M. Sc.

Résumé—Un nouveau champ scalaire est proposé afin de compléter la représentation des variables du champ électromagnétique sous la forme d'un
quadrivecteur. L'utilisation d'un nouveau produit vectoriel mathématique, afin de l'appliquer à quatre vecteurs, donne une nouvelle équation des forces,
qui est proposée pour remplacer l'équation classique des forces de Lorentz. Cette équation de forces évite les difficultés actuelles de l'électromagnétisme
en résolvant certains des paradoxes dits électromagnétiques, sans l'utilisation d'entités cachées.

Termes de l' index —électromagnétisme classique, forces de Lorentz, champs électromagnétiques, quantité de mouvement cachée.

1 PRÉSENTATION
ils ne sont pas covariants sous la LT", ce qui lui permet
L A covariance de Lorentz de l'homogène
Les équations de Maxwell ont été prouvées par d'exprimer son opinion que toute son analyse "suggère
Einstein, dans la section 6 de son article Sur fortement que la violation du principe de relativité et
l'électrodynamique des corps en mouvement [1]. l'existence des paradoxes électrodynamiques proviennent
Cependant, une preuve similaire de la covariance de de l'utilisation de grandeurs 3D", au lieu de grandeurs
Lorentz n'a pas été fournie pour les équations non physiques quantités dans l'espace-temps 4D.
homogènes, qui incluent les densités de charge et de courant.
Contrairement à l'utilisation de grandeurs 3D, lorsque
Engelhardt le confirme en expliquant que « les 'corps des expressions 4D sont utilisées, les résultats sont
en mouvement' porteurs de charges et de courants ne correctsÿ: "Évidemment, l'approche avec des quantités 4D
sont pas traités dans son analyse. Il ne traite que des invariantes de Lorentz donne la réponse relativistement
équations homogènes de Maxwell qui ne conduisent pas correcte d'une manière suffisamment simple et
à des contradictions. [2] transparente" (emphase de l'auteur original) [6 ].

Il s'agit d'une grave lacune qui a été commentée par L'absence de covariance de Lorentz des équations de
certains auteurs, par exemple en ce sens que « Ce résultat Maxwell conduit à la conclusion que nous devrions utiliser
indique que, contrairement à la croyance générale qui des variables 4D, puisque les variables 3D, bien que
prévaut dans l'article fondamental d'Einstein, les covariantes de Lorentz, sont également covariantes
transformations habituelles de E et B ne sont pas galiléennes et, éventuellement, indifférentes aux autres
relativistes correctes. ” [3]. transformations de coordonnées. C'est ce que des auteurs
Le même auteur [4], [5] conclut que les transformations comme Whitney et Kiehn ont révélé à partir des équations
habituelles des champs électriques et magnétiques ne de Maxwell : « La vérité est apparemment celle-ci : les
sont pas les transformations de Lorentz de certaines équations de Maxwell sont tout simplement indifférentes
grandeurs 4D bien définies. Ce qui est plus intéressant, aux transformations de coordonnées. Exprimés sous
dans [3] explique qu'il a prouvé, en utilisant à la fois forme de tenseur/matrice, ils conservent la même forme
l'algèbre géométrique et les formalismes tensoriels, « que sous n'importe quel type de transformation de coordonnées
les équations usuelles de Maxwell avec les 3D E et B physiquement plausible » [7], [8].
changent de forme sous la LT et donc que
La structure du tenseur de champ électromagnétique
• D. Sa´a Emeritus, Département d'Information et d'Informatique, Escuela Polit de second rang F µÿ, avec utilisation exclusive des
´ecnica Nacional, Quito-Equateur. variables 3D E et B, telles que définies en Physique
Courriel : diego.saa@epn.edu.ec
actuelle, voir par exemple [9, Section 11], confirme ce qui
Manuscrit reçu le 18 mars 2015 a été dit plus haut. C'est-à-dire qu'il suggère que son
Machine Translated by Google
MARS 2015 2

la représentation doit être complétée par un quatrième terme avec «ÿun fil porteur de courantÿ» ont à voir avec des
(ou plutôt un premier terme, comme nous l'utiliserons). Notez courants neutres, mais la dérivation contredit cette
que seuls les champs E et B apparaissent dans un tel neutralité.ÿ» [11]
tenseur, mais les densités de charge et de courant ne sont
incluses ni explicitement ni implicitement. Par conséquent, L'équation de Lorentz est la seule acceptée en physique
seules les équations de Maxwell homogènes peuvent être classique pour calculer les forces, impliquant les champs,
construites directement, comme dans les équations (11.142) mais semble avoir quelques lacunes révélées par la théorie
et (11.143) de Jackson [9, Section 11]. Les densités de et les expériences, comme l'effet Aharonov-Bohm, où le
charge et de courant ne peuvent pas être dérivées du tenseur champ magnétique est nul dans la région à l'extérieur du
de champ électromagnétique et doivent être incluses par solénoïde.
d'autres moyens, comme dans l'équation (11.141) de Jackson Donc, comme il n'y a pas de champ B , mais expérimentalement
[9, Section 11]. les charges sont affectées, des physiciens comme Feynman
[12], au lieu de le corriger, se sont crus en droit de le «
La proposition du présent article consiste à définir un remplacer » :
quadrivecteur, qui comprend un champ scalaire s, en plus
des champs E et B habituels , afin de former un quadrivecteur « Nous voudrions maintenant énoncer la loi qui, pour la
électromagnétique complet M, dans le but de remplacer le mécanique quantique, remplace la loi F = qv × B.
tenseur de second rang F µÿ . Ce sera la loi qui déterminera le comportement des particules
Cela s'avérera utile pour générer les équations de Maxwell de mécanique quantique dans un champ électromagnétique.
complètes inhomogènes ou dépendantes de la source, ainsi Puisque ce qui se passe est déterminé par les amplitudes, la
qu'une nouvelle forme d'équation de force. loi doit nous dire comment les influences magnétiques
affectent les amplitudes ; nous n'avons plus affaire à
La physique classique accepte la formule suivante, l'accélération de la particule. La loi est la suivante : la phase
attribuée à Lorentz, pour calculer la force qu'un champ de l'amplitude à arriver par n'importe quelle trajectoire est
électromagnétique produit sur une particule de charge q : modifiée par la présence d'un champ magnétique d'une
quantité égale à l'intégrale du potentiel vecteur sur toute la
trajectoire multipliée par la charge de la particule sur la
F = q (E + v × B) (1) constante de Planck. C'est-à-dire,

Cependant, dans la littérature, nous trouvons quelques q


Changement magnétique de phase = ÿ
objections et aussi quelques tentatives pour corriger cette ¯h Z A·ds. (15.29)
formule de force. Sa modification fut bientôt suggérée par M. (2)
Abraham et PAM Dirac, qui trouvèrent la force dite de « S'il n'y avait pas de champ magnétique, il y aurait une
réaction de rayonnement » induite par l'auto-interaction d'une certaine phase d'arrivée. S'il y a un champ magnétique
particule chargée ponctuellement. [dix] n'importe où, la phase de l'onde qui arrive est augmentée de
l'intégrale dans l'équation. (15.29).
O'Rahilly, [11], après avoir exprimé « La preuve ordinaire Bien que nous n'ayons pas besoin de l'utiliser pour notre
de la formule est extrêmement insatisfaisante » cite Lorentz présente discussion, mentionnons que l'effet d'un champ
disant ce qui suit à propos de la formule, que nous électrostatique est de produire un changement de phase
transcrivons en partie : « Elle est obtenue en généralisant les donné par le négatif de l'intégrale temporelle du potentiel
résultats d'expériences électromagnétiques. Le premier terme scalaire ÿ :
représente la force agissant sur un électron dans un champ
électrostatique... Par contre, la partie de la force exprimée
par le second terme peut être déduite de la loi selon laquelle q
Changement de phase électrique = ÿ
un élément d'un fil parcouru par un courant est actionné par ¯h Z ÿ dt.
un champ magnétique. (3)

O'Rahilly poursuit : « Il y a deux objections écrasantes à cette Ces deux expressions sont correctes non seulement pour les
prétendue généralisation. (1) Les deux « cas particuliers » ici champs statiques, mais donnent ensemble le résultat correct
« combinés » sont tout à fait incompatibles. Dans un cas pour tout champ électromagnétique, statique ou dynamique.
nous avons des charges au repos, dans l'autre les charges C'est la loi qui remplace F = q (E + v × B) » [12]
sont en mouvement ; ils ne peuvent pas être immobiles et en
mouvement. (2) Expériences
Machine Translated by Google
MARS 2015 3

Mansuripur affirme avoir trouvé : "des preuves théoriques a des problèmes qui n'ont toujours pas été résolus.
incontestables de l'incompatibilité de la loi de Lorentz avec
les principes fondamentaux de la relativité restreinte". Il En particulier, ce qui suit semble être d'autres aspects,
conclut : « Nous soutenons que la loi de Lorentz doit être en plus de l'équation des forces de Lorentz, qui semblent
abandonnée au profit d'une expression plus générale de la être incorrects dans la théorie électromagnétique classique
densité de force électromagnétique. [13] et, espérons-le, seraient corrigés avec notre théorieÿ:

Mansuripur explique qu'une équation différente pour les Les transformations du champ classique pour E et B ne
forces pourrait résoudre le problème : « Je soutiens que, si révèlent apparemment aucun effet Doppler, sous forme de
Shockley avait utilisé une loi de force alternative proposée décalage vers le rouge ou de décalage vers le bleu, lorsqu'un
en 1908 par Albert Einstein et Jacob Laub, il aurait trouvé boost est produit dans la direction du mouvement, puisque
que la combinaison des équations macroscopiques de les variables correspondantes ne sont pas modifiées dans
Maxwell et de la La loi Einstein-Laub est conforme aux lois cette direction.
de conservation sans avoir besoin d'entités cachées. [14].
Sa proposition, d'utiliser une théorie différente pour calculer De plus, les invariants de champ ne sont apparemment
les forces électromagnétiques au lieu des forces de Lorentz, pas préservés dans les applications élémentaires, comme
a été soumise à de nombreuses attaques par ceux qui dans le paradoxe de Jefimenko, considéré plus loin dans cet
proposent des "entités cachées" afin de résoudre les article.
problèmes de la théorie électromagnétique classique.
En outre, certains auteurs [19] remarquent que le vecteur
de Poynting pour un circuit filaire ne prédit pas le flux
d'énergie parallèle au fil lui-même, concluant que "Ceci et
Cependant, les physiciens qui proposent l'utilisation de d'autres caractéristiques imprévues de certains flux de
variables cachées semblent méconnaître sa localisation Poynting incitent certains auteurs à définir un vecteur de
précise et son existence, ce qui est confus et douteux. Par Poynting en utilisant S = (E × B) + ÿ1µ0
× _X
champ
[Éq. (15.36)].
vectorielLe
X est
exemple, l'auteur dans [15] dit qu'il a "montré qu'en plus de choisi pour faire pointer (15.36) dans une direction plus
la quantité de mouvement cachée, une fausse quantité de "naturelle".
mouvement 4 transporte de l'énergie cachée". Griffiths
explique que « localiser cet élan caché peut être subtil et Enfin, l'article d'Engelhardt [2] révèle brillamment qu'« il
difficile ». [16] existe un´ moyen direct de montrer que le résultat de Lienard-
Babson et. al.ÿ: «ÿUne caractérisation définitive du Wiechert, qui conduit aux champs [obtenus en remplaçant
phénomène reste insaisissable, et certains ont suggéré que les potentiels de Lienard-Wiechert ´ dans les expressions
le terme devrait être élargi pour inclure toutes les contributions classiques des champs E et B en termes de potentiels], ne
strictement relativistes à la quantité de mouvementÿ; d'autres satisfait que les équations homogènes de Maxwell, mais les
ont demandé que le terme soit complètement supprimé. [17] inhomogénéités ne sont pas correctement prises en compte.

Franklin [18] contre Babson et.al. résultats, en disant que


"Nous avons vu qu'aucun des mécanismes proposés ne
conduit à un élan caché." et conclut: «Notre conclusion est À la lumière de cette dernière conclusion, il semble être
que le théorème du« centre d'énergie »ne s'applique pas à une bonne propriété que la théorie proposée dans les lignes
l'impulsion EM d'une distribution de courant de charge suivantes permette de conclure que les variables
statique, et que« l'impulsion cachée »n'est ni nécessaire ni électromagnétiques (par exemple les champs électriques et
présente dans la distribution de courant de charge. La force magnétiques, les potentiels scalaires et vectoriels et les
externe nécessaire pour maintenir la matière au repos lors courants de charge) satisfont juste homogène ( D'Alembert)
de la création de la distribution de courant de charge va équations d'onde, bien que cette propriété ne soit pas
directement dans l'élan EM sans déplacer la matière ni démontrée dans le présent article.
cacher l'élan.

2 QUATRE VECTEURS ET MAXWELL


Donc, tout cela laisse entendre que la formule de force ÉQUATIONS
de Lorentz ne fonctionne pas très bien et jette le soupçon Dans cette section, nous allons faire un bref résumé de la
que la théorie électromagnétique actuelle façon de développer l'électromag-
Machine Translated by Google
MARS 2015 4

théorie nétique. Dans des articles antérieurs, le présent auteur La description des champs électromagnétiques en
a proposé la formule suivante pour obtenir le produit de quatre Physique actuelle, en tant que tenseur de second rang, semble
vecteurs. Supposons deux quadrivecteurs génériques Aµ = (a, quelque peu déficiente, puisqu'elle nécessite six paramètres
a) et Bµ = (b, b) où le premier terme de chaque quadrivecteur différents pour représenter les champs électriques et
est un scalaire et le second terme, en gras, est un vecteur 3D. magnétiques dans chacune des trois directions de l'espace :
(Ex, Ey, Ez , Bx, Par, Bz). Cependant, ces champs peuvent
Le sous-indice ou super indice dans certains quatre vecteurs être représentés en termes de potentiels, qui n'ont besoin que
n'est qu'une question de notation, afin de se distinguer des de quatre paramètres : (ÿ, Ax, Ay, Az). Il y a donc un excès de
autres variables. Le conjugué d'un quadri-vecteur Aµ, noté par deux paramètres dans le tenseur de Faraday F µÿ
un surlignement, Aµ, change les signes de la partie vectorielle : .
(a, ÿa). De plus, dans notre expression précédente 9 apparaît un
Le produit à quatre vecteurs est calculé avec la formuleÿ: terme scalaire (premier terme du quadrivecteur), qui n'a pas
été reconnu par la Physique actuelle.
Pour résoudre ces lacunes, notre proposition est de représenter
le champ électromagnétique à l'aide du quadrivecteur
Aµ B µ = (ab + a · b, ÿab + ab + a × b) (4) complexe, qui dans notre théorie remplit les mêmes fonctions
Où · et × représentent les produits vectoriels scalaires et que le tenseur de Faradayÿ:
croisés classiques.
Il existe un produit alternatif, où les signes des deux premiers M = (s, je E + c B) (dix)
termes de la partie vectorielle sont intervertis.
Chaque position dans la partie spatiale de ces quatre
vecteurs est une variable complexe unique, constituée du
champ électrique et magnétique correspondant dans la
Afin de dériver les formules de l'électromagnétisme, nous
direction des coordonnées respectives.
devons définir l'opérateur à quatre gradientsÿ:

1ÿ ÿ ÿ Il ne s'agit plus maintenant que de mettre cela en équation


ÿµ = ( - je , , ÿ) (5)
c ÿt ÿx, ÿy ÿz avec le quadrivecteur électromagnétique 9 obtenu ci-dessus,
avec lequel on obtient les définitions connues du
Ou, en abrégeant les trois dérivées spatiales avec le
champs électriques et magnétiques en termes de potentiels,
Opérateur hamiltonien ÿÿ:
plus l'apparition du terme scalaire, qui n'a pas d'équivalent
1ÿ dans la Physique actuelle, que nous avons appelé s :
ÿµ = ( - je , ÿ) (6)
c ÿt

Définissons également le quadrivecteur potentiel sous la 1 ÿÿ


forme : s=- ÿ ÿ · UNE (11)
c ÿt

A µ = ( ÿ iÿ, ÿA) (sept)


Ceci définit le scalaire électromagnétique.
Sa forme est identique à la jauge de Lorenz. Cette « jauge
Où les signes curieux sont choisis pour se conformer à la » est normalement supposée égale à zéro en Physique
convention habituelle pour les champs électromagnétiques, actuelle. Cependant, si tel était le cas, le scalaire
comme indiqué ci-dessous. électromagnétique serait nul et, selon la théorie actuelle, on
Ensuite, les champs électromagnétiques sont calculés en obtiendrait les équations de Maxwell homogènes. Cela signifie,
prenant le quadrigradient du quadrivecteur potentielÿ: toujours dans la théorie actuelle, que la jauge de Lorenz revient
à supposer que les densités de charge et de courant sont
M = Aµ ÿµ (8) nulles.

En développant cette expression avec l'utilisation de la


formule du produit (4):
En plus de la définition donnée ci-dessus pour le scalaire
M=- 1 ÿt
ÿÿ
c
ÿ ÿ · UNE , électromagnétique s, l'égalité des expressions 9 et 10 fournit
également les définitions classiques des champs électriques
1 ÿA et magnétiques en termes de potentielsÿ:
-je ÿ je ÿÿ +
c ÿt

1 ÿA
ÿ×A (9) E = ÿ ÿ ÿÿ (12)
c ÿt
Machine Translated by Google
MARS 2015 5

cB = ÿ × A (13) Pour faciliter la comparaison, nous reproduisons dans la


ligne suivante la densité de force de Lorentz, équation 19, telle
Ensuite, le lecteur intéressé pourrait souhaiter développer
que donnée par Jackson dans la section 12.10, équation
les quatre équations de Maxwell en assimilant à zéro le quatre
(12.121) [9, p.557, p.611]
gradient du quatre vecteur électromagnétique sous la forme
fournie dans l'équation. 10, soit :
1 11
fÿ ÿ
F ÿÿJÿ = ( J · E, ÿ E + J×B) (19)
ÿµ M = 0 (14) c c c

Cette dérivation des équations de Maxwell est beaucoup plus


La différence la plus importante entre notre formule de forces
simple que celle utilisée dans la Physique actuelle.
18 et la classique, 19, est que nous avons dérivé, dans la
Mais nous n'avons pas besoin de dériver les équations de composante imaginaire de la partie vectorielle, la nouvelle
force ÿsJ, proportionnelle au courant et au scalaire
Maxwell dans le présent article, veuillez donc vous référer
électromagnétique.
ailleurs [20] pour les détails.

Le quadrivecteur densité de courant est le classique [9,


Eq. (11.128)] et est obtenu en prenant le gradient du seul 4 COMMENT LES CHAMPS ÉLECTROMAGNÉTIQUES
champ scalaire électromagnétique, s, mais est ici supposé TRANSFORMER
comme étant donnéÿ: La composante réelle dans la partie vectorielle de 18, c'est-à-dire
J µ = (iÿ c, J) (15) ( ÿ c2 B ÿ J × E), remonte à la première partie de la seconde des
équations de Jackson (11.149), ci-dessous numérotée ici 21, qui
donne la transformation de le champ magnétique B d'une trame
3 LES FORCES ÉLECTROMAGNÉTIQUES
à l'autre :
QUATRE VECTEURS
Les forces électromagnétiques doivent être calculées en 0 2
E ÿ = ÿ( E + ÿ~ × B) ÿ ÿ~(ÿ~ · E ) ÿ + 1 (20)
multipliant non seulement la charge, mais les quatre courants,
par le quadrivecteur électromagnétique. Par conséquent, le
quadrivecteur des forces électromagnétiques est donné par le
0 2
B ÿ = ÿ( B ÿ ÿ~ × E) ÿ ÿ~(ÿ~ · B ) ÿ + 1 (21)
produit
F µ = JµM _ (16) Le dernier terme de l'équation 20 devrait donner un terme
proportionnel au courant, J, lorsqu'il est multiplié par la charge
Où nous supposons que le quatre vecteur
électromagnétique est sous la forme ou la densité de charge, de la même manière que le terme J ×
B peut être produit sur la base du terme contenant le produit
croisé (ÿ ~ × B).
M = (s, je E + c B) (17)
Cependant, il n'y a aucune preuve d'une telle force dans
Et les champs incluent déjà le facteur gamma l'équation classique des forces correspondante.
correspondant, le cas échéant. Une des raisons de la disparition de ce terme doit être attribuée
Alors, le quadrivecteur des forces électromagnétiques est donné au fait que l'électrodynamique classique traite les champs
par le produit F µ = i Jµ M. Cettecelle
équation
apparaissant
est essentiellement
sous les électromagnétiques comme des vecteurs tridimensionnels ;
équations (11.144) ou (12.118) dans le livre de Jackson [9, p.557, une autre raison est que l'électrodynamique classique n'exige
p.611] . L'unité imaginaire est utilisée pour se rapprocher de pas un tel terme dans l'équation classique des "forces de
certains signes dans les références et, comme dans les équations Lorentz".
précédentes et suivantes, n'est qu'un artefact mathématique qui Par conséquent, les derniers termes des équations 20 et 21
fonctionne bien pour garder une trace des différentes composantes ne sont utilisés que pour préserver les invariants de champs
des quatre vecteurs, comme le sont les équations électriques et B2 ÿ E2 et B · E, mais pas pour générer l'équation de force de
champs magnétiques. Lorentz. Cependant, notez que, comme le montre notre
équation 19 ci-dessus, Jackson a l'intention de considérer les
forces comme un quadrivecteurÿ; mais, puisque le scalaire
Ensuite, le quadrivecteur de densité de force est électromagnétique n'existe pas en Physique classique, la force
F µ = ( (ÿ cs ÿ J · E) + ic J · B, ( ÿ c2 B ÿ
scalaire correspondante ne peut pas être obtenue par un
produit direct de la charge ou de la densité de charge multipliée
J × E) + je(ÿs J + ÿ c E + c par les intensités de champ.
J × B) ) (18)
Machine Translated by Google
MARS 2015 6

D'autre part, examinons la forme de nos transformations métamorphoses.


de champs électromagnétiques, qui conduisent directement
aux forces, lorsqu'elles sont multipliées par la charge ou la Il est vrai, comme cela a été exprimé dans la référence
densité de charge [21], que la force de Lorentz doit être contenue dans 20 et
0
21, pour la raison suivante : La force sur une charge au repos
s = ÿ( s ÿ ÿ~ · E + je ÿ~ · B) (22) par rapport aux sources dans les équations de Maxwell est
connue pour être qE. Lorsque la charge se déplace, les
0 champs électriques et magnétiques dans le référentiel de
E = ÿ( ÿ sÿ~ + E + ÿ~ × B) (23)
repos de la charge peuvent être obtenus en transformant les
champs selon 20 et 21, ce qui devrait donner la force de
0

B = ÿ( B ÿ ÿ~ × E) (24) Lorentz 19. Si la particule se déplace à vitesse constante, on


peut toujours définir un système de coordonnées qui se
Après avoir replacé ces champs dans le quadrivecteur déplace avec la particule, de sorte que la force électrique ne
électromagnétique et l'avoir quadrillé, tout cela se simplifie en change pas et que la force magnétique s'annule dans
invariants de champ sous la forme l'équation classique des forces de Lorentz.
2 2
2s +B ÿE + 2iB · E (25)

A titre d'exemple, appliquons les expressions de Jackson


20 et 21 pour calculer les transformations de champ d'un
champ électrique dans le x 5 LE PARADOXE DE JEFIMENKO
direction, appelons-la Ex, avec tous les champs restants Jefimenko [22] a considéré une particule de charge q2 au
égaux à zéro, soumis à une impulsion de magnitude ÿx dans repos sur l'axe des x, tandis qu'une autre particule de charge
la même direction x . Le résultat est que ce champ n'est pas identique, q1, se trouve sur l'axe des x, avec sa vitesse, v1,
affecté par les transformations, ce qui semble suspect. pointant le long de l'axe des x, comme dans le chiffre suivant :

v1
Par contre, dans notre théorie appliquons l'équation 23 ; q1 q2
supposons que le champ soit à l'origine et que le champ
scalaire s soit égal à Ex, ce qui rend le champ invariant égal Le champ électrique relativiste de la particule 1 à la
à zéro, et que l'observateur s'éloigne de ce champ vers la position de la particule 2 est donné par Jackson [9, Section
droite. Le résultat est un redshift de la forme 11], [Eq. (11.152), avec le paramètre d'impact b égal à zéro]
et utilisé également par Jefimenko [22], Caprez [23] et
d'autresÿ:

Ex s 1 ÿ ÿx (26)
1 + ÿx E12 = 2 2 ÿ vkq1
11
xˆ (28)
t2
Mais, si l'observateur s'approche du champ
à partir de la gauche, puis il y a un blueshift de la forme
2
v1

Où ÿ1 = 1/ q 1 ÿ vitesse l'indice 1 fait référence à la


2,c

Ex s 1 + ÿx (27)
1 ÿ ÿx de la particule 1, et xˆ est un vecteur unitaire dans la direction
de l'axe des coordonnées joignant les charges.
Pour obtenir ce dernier résultat, il faut intervertir les signes
des termes contenant s dans les équations 22 et 23 (ces
Le problème avec cette équation est que les champs
équations sont obtenues avec application du produit alternatif,
électriques et magnétiques ne devraient pas changer lorsqu'ils
au lieu de 4). De plus, si le signe de ÿx est modifié, le décalage
sont perçus par un observateur dans la direction du
vers le rouge et le décalage vers le bleu sont également
mouvement, comme prescrit par la transformation classique
interchangés. Tout cela est en accord avec les décalages
des champs 20 et 21, mais cela n'est pas satisfait par ce que
vers le rouge et les décalages vers le bleu relativistes des
le précédent L'expression de Heaviside propose.
champs.
Alors que le champ électrique de la particule 2 à la
position de la particule 1 est donné par
Ainsi, il devrait être assez évident que les derniers
résultats semblent plus complets, raisonnables et bien kq2
meilleurs que celui obtenu avec la méthode classique E21 = ÿ
2
contre 1 xˆ (29)
t2
Machine Translated by Google
MARS 2015 sept

Lorsqu'ils sont multipliés par la charge de la particule 6 COMMENT APPLIQUER LES ÉQUATIONS
affectée, ces champs, 28 et 29, produisent deux forces qui ne
Essayons de résoudre le paradoxe de Jefimenko avec l'utilisation
forment pas une paire de Newton [23].
de nos équations proposées.
Supposons que le champ électrique de la charge q1, considérée
au repos dans son propre référentiel, soit E12, à la distance où
Pour contrebalancer cet écart, les auteurs classiques
se situe la charge q2 .
obtiennent essentiellement la différence entre 28 et 29, via le
En supposant que le champ magnétique, B, est nul et que
potentiel vecteur, et l'incluent en quelque sorte dans le système
l'invariant électromagnétique de cette charge, 25, est nul, alors
analysé, de telle sorte que la somme des impulsions est nulle,
s12 devient égal à E12. Par conséquent, le quadrivecteur
ce qui semble incorrect.
électromagnétique dans ce cas est
Avec cet ajout, le 3-momentum du système en mouvement est
(E12, iE12, 0, 0) (30)
remis à zéro, comme s'il s'agissait d'un système statique, malgré
le fait qu'il se déplace maintenant. Remplacer ces valeurs dans les équations 22-24, ce qui
Mais ce n'est pas la seule préoccupation, puisqu'il est bien connu revient à faire un boost du quadrivecteur champ électromagnétique
que l'amplitude du 4-momentum est la variable qui doit être par une vitesse ÿÿ, puisque la charge q2 doit percevoir le champ
préservée, pas l'amplitude du 3-momentum isolé. électrique comme décalé vers le bleu

(ÿ(E12 + ÿE12), iÿ(E12 + ÿE12), 0, 0) (31)


Il y a aussi une contradiction assez nette entre les équations
Le premier terme est le champ scalaire amplifié et le second
utilisées, puisque ce déséquilibre n'aurait pas dû se produire en
terme est le champ électrique amplifié dans la direction x ; qui,
premier lieu si l'équation de force de Lorentz classique 19 ainsi
que la transformation classique des champs 20 et 21 avaient été comme on peut le voir, est soumis à un blueshift Doppler
utilisées, puisque dans ce cas les deux les champs électriques relativiste.
et magnétiques ne sont pas modifiés par les transformations du
champ classique dans la direction du mouvement. Alors, pourquoi Le facteur constituant la différence entre ce champ électrique
le champ électrique 28 contient-il maintenant un facteur ÿ au amplifié et l'original (statique)
carré ? l'un est
(ÿ + ÿÿ ÿ 1) (32)

Ce facteur peut aussi s'écrire sous la forme


De plus, pour que ces équations soient raisonnables, elles
doivent conserver l'invariant électromagnétique relativiste B2 ÿ ÿ ÿ 1 + p ÿ ÿ 1,2 et semble posséder des propriétés
.
E2 En supposant la présence d'une charge statique isolée intéressantes, qui pourraient être étendues aux corps
comme q2, son invariant relativiste doit être juste E2 puisqu'il n'a macroscopiques.
pas de champ magnétique. Mais cet invariant devrait être 21,
préservé par tous les champs perçus par un observateur se
déplaçant à vitesse constante dans la direction de cette charge.
7 CONCLUSION
Mais, pour le cas de la charge q1, qui est identique à q2, les
équations classiques de transformation des champs L'ajout apparemment innocent d'un nouveau champ scalaire,
électromagnétiques ne produisent aucun effet dans la direction afin de compléter le quadrivecteur électromagnétique, s'avère
du mouvement. Par conséquent, le seul champ connu pour cette transformer radicalement la théorie électromagnétique. En
charge, qui est E12, devrait conserver l'invariant. Mais une telle résumé, tous les changements survenus semblent corriger des
conservation paraît impossible, puisque les physiciens classiques problèmes permanents, comme dans l'équation des forces de
s'obstinent à considérer le champ électrique d'une charge en Lorentz, dans les transformations des champs, qui incluent
mouvement, comme E12 ci-dessus, comme dépendant de la désormais les effets Doppler, dans la prise en compte des
invariants du champ et dans la propriété que les variables
vitesse, à l'intérieur du facteur gamma.
électromagnétiques sont maintenant nécessaire pour satisfaire
uniquement des équations d'onde homogènes.

Les praticiens de l'électricité et de l'électronique, comme


Par conséquent, il semble que l'électrodynamique classique ceux de l'Institute of Electric and
ics gère incorrectement les champs électromagnétiques. Les ingénieurs électroniciens, semblent avoir nul ou très
Machine Translated by Google
MARS 2015 8

intérêt limité à corriger la théorie qu'ils utilisent. [16] Griffiths, D. Resource Letter EM-1: Electromagnetic Mo mentum Am.
Par exemple, après avoir été soumis aux Actes de l'IEEE et J.Phys. 80 (1), 7-18, (2012)
[17] Babson, D., et al. Moment caché, moment de champ et impulsion
également aux Transactions de l'IEEE sur les antennes et la
électromagnétique, Am. J. Phys., 77, n° 9, 826-833, (2009)
propagation, les éditeurs ont répondu que "Malheureusement,
l'article ne convient pas à la publication dans ces Transactions [18] Franklin, J. Le moment électromagnétique des distributions de courant
car le sujet sera (sic) de portée limitée. intérêt pour les lecteurs de charge statique, American Journal of Physics 82 (9) , 869-875
(2014)
de la revue.
[19] Zangwill, A. Modern Electrodynamics, Cambridge University Press,
2013.
[20] Saa, D. Quatre vecteurs en électromagnétisme, Revista Politecnica,
Le scientifique en général devrait également remarquer 28 (2009), 1, pp.121-145. Disponible en ligne sur http://
www.academia.edu/8260954/Maxwells equations from four-vectors
que les transformations proposées du champ électromagnétique
pourraient avoir un impact sur la théorie de la relativité, [21] Engelhardt, W. Sur la transformation relativiste des champs
puisque l'origine des «transformations de Lorentz» relativistes électromagnétiques, Apeiron, 11, 2, (2004)
peut leur être attribuée. [22] Jefimenko, OD Un paradoxe relativiste qui viole apparemment la loi
de conservation de la quantité de mouvement dans les systèmes
électromagnétiques, Eur. J.Phys. 20 (1999) 3944.
[23] Caprez A., Batelaan H. Feynman's Relativistic Electrodynamics
Paradox and the Aharonov-Bohm Effect, Found.
RÉFÉRENCES Phys. (2009) 39ÿ: 295.

[1] Einstein, A. On the Electrodynamics of Moving Bodies, Annalen der


Physik, 30 juin 1905. Traduction anglaise par John Walker, http://
www.fourmilab.ch/ [2] Engelhardt, W., Gauge Invariance in Classical
Electrodynamics, Annales de la Fondation Louis de Broglie, 30 (2005), 2,
pp. 157–178 Fondements de la physique, 36, 10, (octobre 2006).

[4] Ivezic, T. La différence entre les transformations standard et de Lorentz


des champs électriques et magnétiques. Application à Motional EMF,
Foundations of Physics Letters 18, 4, (août 2005).

[5] Ivezic, T. Lorentz et Transformations "apparentes" des champs


électriques et magnétiques, Fizika A (Zagreb) 17, 1 (2008).
[6] Ivezic, T. The Proof that Maxwell Equations with the 3D E and B are not
Covariant on the Lorentz Transformations but on the Standard
Transformations: The New Lorentz Invariant Field Equations,
Foundations of Physics, 35, 9, (septembre 2005) .

[7] Whitney, théorie de C. Maxwell et relativité galiléenne, Pro


actes du NPA, 7, 1, (2010).
[8] Kiehn, RM Systèmes hors équilibre et processus irréversibles, Université
de Houston, (2004), p. 172.
[9] Jackson, JD Électrodynamique classique, 3e. Édition, John Wiley &
Sons, Inc. (1999). p.557, p.611.
[10] Yaghjian, AD Dynamique relativiste d'une sphère chargée : mise à jour
du modèle de Lorentz-Abraham, Springer-Verlag (1992)

[11] O'Rahilly, A. Théorie électromagnétique, Dover Publications


Inc., New York, (1965), p. 561.
[12] Feynman, RF Les conférences Feynman sur la physique, Vol. II
Ch.15, Addison-Wesley (1964).
[13] Mansuripur, M. Problème avec la loi de la force de Lorentzÿ: en
compatibilité avec la relativité restreinte et la conservation de la
quantité de mouvement, Phys. Rév. Lett. 108, 193901 (2012)
[14] Mansuripur, M. La loi de force de l'électrodynamique classiqueÿ:
Lorentz contre Einstein et Laub, Piégeage optique et micromanipulation
optique X, édité par K. Dholakia et GC Spalding, Proc. de SPIE 8810,
88100K-1:18 (2013).
[15] Comay, E. Exposing “hidden momentum”, American Journal of Physics
64 (8), 1028-1034, (1996)

Afficher les statistiques de publication

You might also like